June 2002, PrepTest 37, Game 4 (19-24) Forum

Prepare for the LSAT or discuss it with others in this forum.
Post Reply
Bigsby

New
Posts: 71
Joined: Sat Jan 15, 2011 2:03 pm

June 2002, PrepTest 37, Game 4 (19-24)

Post by Bigsby » Wed Jan 19, 2011 8:56 pm

This question revolves around "a swim team with exactly five members- J, K, L, M, O-......"

I was wondering what you guys did for a setup on this. I seem to not be able to make many deductions on this game and even though I finished within the time, I made a guess out of 3 choices for 22 (between B, C, and D) and got it correct and got number 23 wrong (I put down A, and see how it is wrong).

I felt like, for these, I was lacking a good Master Diagram and thus had to take my time more and methodically going through the remaining contenders and made me more inefficient/mistake-prone. Any of you LSAT Jedis (Jedis's? Jedi?) willing to help?

JG7773

Bronze
Posts: 333
Joined: Mon Mar 22, 2010 2:02 am

Re: June 2002, PrepTest 37, Game 4 (19-24)

Post by JG7773 » Wed Jan 19, 2011 10:49 pm

Bigsby wrote:This question revolves around "a swim team with exactly five members- J, K, L, M, O-......"

I was wondering what you guys did for a setup on this. I seem to not be able to make many deductions on this game and even though I finished within the time, I made a guess out of 3 choices for 22 (between B, C, and D) and got it correct and got number 23 wrong (I put down A, and see how it is wrong).

I felt like, for these, I was lacking a good Master Diagram and thus had to take my time more and methodically going through the remaining contenders and made me more inefficient/mistake-prone. Any of you LSAT Jedis (Jedis's? Jedi?) willing to help?

--- --- --- --- ---
1/6 2/7 3/8 4/9 5/10
M>O(1) / Never: KL / At least 1 OJ / J Never 4/9

Hypo: J ___ M ____ O

For #22 - A is impossible because of Rule 2 (Jacobson can't swim 4/9)
B - is correct. I found this from a diagram of question 20.
C- Would force a M O J K L which violates Rule 1
D - Violation of Rule 3 (Miller would swim 5/10 and Ortiz wouldn't be able to have their first lap after Miller)
E - Violation of Rule 3 (Ortiz would swim 1/6 and wouldn't have their first lap after Miller)

For #23 - A - Should be thrown out because question 19 shows that Jacobson can swim 1/6
B - If you put J in 2 you would be forced to put O is 1 that would violate the first O coming after M
C - Question 20 should of had a sketch with J in 3/8
D - If he can't swim one, he can't swim 6
E -In question 21, we had Jacobson in 5, therefore he would also be in 10.

Hope this helps.. any questions let me know.

Post Reply

Return to “LSAT Prep and Discussion Forum”